You are on page 1of 7

MATH 112 Homework 4

Solutions

1. Find all the values of the constant p > 0 for which the following series are convergent:
X∞ X∞
1 (n!)p
a) b)
n=3
n ln n (ln(ln n))p n=1
(3n)!

Solution.
a) Let f (x) = 1/(x ln x (ln(ln x))p ), x ≥ 3. Since f (x) is continuous, positive and
decreasing on [3, ∞), we can use the Integral Test.
Z ∞ Z ∞
1 1
p
dx = p
dp (u = ln(ln x) substitution).
3 x ln x (ln(ln x)) ln(ln 3) u

Since ln(ln 3) > 0, the integral on the right is convergent if and only if p > 1. So the given
series is convergent if and only if p > 1.
(n!)p
b) an = (3n)!
> 0 for all n. We use the Ratio Test.
1

p if p = 3 
an+1 (n + 1) 27
ρ = lim = lim = 0 if p < 3
n→∞ an n→∞ (3n + 1)(3n + 2)(3n + 3) 
∞ if p > 3

So the given series is convergent if and only if p ≤ 3.

2. Find

the exact sum of the series: ∞
X 2n + 3n cos(πn) X n
a) b)
n=1
5n n=1
(n + 1)(n + 2)(n + 3)

Solution.
a) First note that cos(πn) = (−1)n , so by the geometric series sum formula
X∞ X∞
2n + 3n cos(πn) 2n + 3n (−1)n
=
n=1
5n n=1
5n
X∞ µ ¶n X∞ µ ¶n
2 3
= + −
n=1
5 n=1
5
1 1 2 3 7
= 2 −1+ 3 −1 = − = .
1− 5
1 − (− 5 ) 3 8 24

b) We use the method of partial fractions and get


µ ¶
n − 12 2 − 32 1 1 4 3
= + + =− − + .
(n + 1)(n + 2)(n + 3) n+1 n+2 n+3 2 n+1 n+2 n+3
Then



µ ¶ µ ¶ µ ¶
1 1 4 3 1 4 3 1 4 3
sn = − − + + − + + − +
2
 2 3 4 3 4 5 4 5 6

| {z } | {z } | {z }
a1 a2 a3
+ ··· 

µ ¶ µ ¶


1 4 3 1 4 3
+ − + + − +
n n+1 n+2 n+1 n+2 n+3 
| {z } | {z }


an−1 an
µ ¶
1 1 4 1 3 4 3
= − − + + − + ,
2 2 3 3 n+3 n+2 n+2

where the cancelation pattern for three consecutive parentheses is as follows:

( − + /)+( −/+ )+(/ − + ).

So µ ¶ µ ¶µ ¶
1 1 4 1 1 1 1
s = lim sn = − − + = − − = .
n→∞ 2 2 3 3 2 2 4

In exercises 4 and 5, test the given series for convergence or divergence. In each case give
reason(s) for your decision.
X∞ X∞
n3 + 5n 1
4. a) b)
n=1
2n n=2
(ln n)3

Solution.
3
a) an = n 2+5n
n > 0 for all n. We use the Ratio Test.

an+1 (n + 1)3 + 5(n + 1) 2n (n + 1)3 + 5(n + 1) 1


ρ = lim = lim n+1 3
= lim 3
= < 1.
n→∞ an n→∞ 2 n + 5n n→∞ 2(n + 5n) 2
P∞ n3 +5n
So by the Ratio Test the series n=1 2n
is convergent.

b) an = (ln1n)3 > 0 for all n ≥ 2. We have that ln n ≤ 3 n for all n ≥ N for some
P
positive integer N . Thus an ≥Pn1 . Since the series ∞ 1
n=1 n is divergent, by the Direct
Comparison Test, the series ∞ 1
n=2 (ln n)3 is divergent.

∞ ∞ √ √
X ¡ ¢ X n+1− n
πn
5. a) e sin π −en b)
n=1 n=1
n+1

Solution.
a) an = eπn sin (π −en ) > 0 for all n. We use the Limit Comparison Test, where bn =
eπn π −en . As n → ∞, we have that π −en → 0. Since limx→0 sinx x = 1, we get

an eπn sin (π −en ) sin (π −en )


L = lim = lim = lim = 1 and 0 < L < ∞.
n→∞ bn n→∞ eπn π −en n→∞ π −en
The series ∞ ∞ ∞ µ π ¶n
X X X e
πn −en
bn = e π =
n=1 n=1 n=1
πe
π e e π
is a geometric series with
P r = e /π . Since π < e , we have r > 1. Since |r| > 1,
the
P∞geometrical series bn is divergent. So by the Limit Comparison Test, the series
πn −en
n=1 e sin (π ) is divergent.
Remark. Why is π e < eπ ?
To show this, consider f (x) = lnxx , x > 0. Show that f (x) has absolute maximum at
x = e. Thus lnππ < lnee . By cross multiplying and exponentiating we get the claim.
√ √ √ √ √ √
n+1− n n+1− n n+1+ n 1
b) an = = √ √ = √ √ > 0.
n+1 n+1 n+1+ n (n + 1) ( n + 1 + n)

1 1 1 P
We have an ≤ √ =. Let b n = . Then 0 < a n ≤ bn for all n, and bn is
n n n3/2 n3/2
convergent (p-series with p = 23 > 1), so by the Direct Comparison Test, the series
P∞ √n+1−√n
n=1 n+1
is convergent.

In exercises 6 and 7, determine whether the given series is absolutely convergent, condi-
tionally convergent or divergent. In each case give reason(s) for your decision.
X∞ µ ¶ X∞ µ ¶n
n 1 n n
6. a) (−1) arctan b) (−1)
n=1
2n + 1 n=1
n+1
µ ¶
n 1 ¡ 1
¢
Solution. a) an = (−1) arctan and |an | = arctan 2n+1 . Let bn = n1 . We
2n + 1
1
have (after the substitution x = 2n+1 and using limx→0 arctan
x
x
= 1)
¡ 1
¢
|an | arctan 2n+1 arctan x arctan x 1 − x 1
L = lim = lim 1 = lim 2x = lim · = and 0 < L < ∞.
n→∞ bn n→∞ x→0 x→0 x 2 2
n 1−x
P P1 P
Since bn = n
is divergent, by the Limit Comparison Test |an | is divergent.
P ¡ ¢
On the other hand, the series ∞n=1 (−1)
n 1
arctan 2n+1 is an alternating series with un =
1 1
arctan 2n+1 . We have that as n increases, { 2n+1 } decreases and so {un } is decreasing
P ¡ ¢
and limn→0 un = 0. So by the Alternating Series Test, ∞ 1
(−1)n arctan 2n+1 is
P∞ n
¡ 1
¢ n=1
convergent. So the series n=1 (−1) arctan 2n+1 is conditionally convergent.
¡ n ¢n
b) The series is an alternating series with un = n+1 . We have that
µ ¶n
n 1 1 1
lim un = lim = lim ¡ n+1 ¢n = lim ¡ ¢ n = 6= 0.
n→∞ n→∞ n+1 n→∞ n→∞ 1 + 1 e
n n

So the Alternating Series Test cannot be used. But because of this fact
µ ¶n
n n
lim an = lim (−1) = lim (−1)n un does not exist.
n→∞ n→∞ n+1 n→∞
P ¡ n ¢n
So by the n-th Term Test, the series ∞ n=1 (−1)n
n+1
is divergent.


X ∞
X
(sin(1/n))(sin n) (−1)n n112
7. a) b)
n=1
n n=1
(n + 1)!

Solution. P
a) an = (sin(1/n))(sin
n
n)
and |an | = | sin(1/n)|
n
| sin n|
≤ (1/n)·1
n
= n12 . Since ∞ 1
n=1 n2 is convergent,
P∞ P
by the Direct Comparison Test n=1 |an | is convergent, that is ∞ n=1
(sin(1/n))(sin n)
n
is
absolutely convergent.
(−1)n n112 n112
b) an = (n+1)!
and |an | = (n+1)!
. We use the Ratio Test

|an+1 | (n + 1)112 n112 + 112n111 + · · · + 112n + 1


ρ = lim = lim 112 = lim = 0.
n→∞ |an | n→∞ n (n + 2) n→∞ n113 + 2n112
P∞
Since 0 ≤ ρ < 1, by the Ratio Test the series n=1 |an | is convergent, that is
P∞ (−1)n n112
n=1 (n+1)! is absolutely convergent.

8. Find the interval of convergence of the following power series. Do not forget to check
the end points:
X∞ X ∞
2 · 4 · · · (2n) (x + 3)n
a) xn b)
n=1
1 · 3 · · · (2n − 1) n=2
n ln n

∞ ¯
X
¯
¯ 2 · 4 · · · (2n) ¯
Solution. a) We apply the Ratio Test to the series ¯ n¯
¯ 1 · 3 · · · (2n − 1) ¯.
x
n=1
¯ ¯
¯ 2·4···(2n)(2n+2) n+1 ¯
¯ 1·3···(2n−1)(2n+1) x ¯ 2n + 2
ρ = lim ¯ ¯ = lim · |x| = |x|.
n→∞ ¯ 2·4···(2n) n ¯ n→∞ 2n + 1
x
¯ 1·3···(2n−1) ¯

The power series converges absolutely if ρ = |x| < 1, and diverges if ρ = |x| > 1.
The end points |x| = 1, that is x = 1 and x = −1 must be checked independently.
P
When x = 1, the series becomes ∞ 2·4···(2n)
n=1 bn where bn = 1·3···(2n−1) . Since bn > 1, limn→∞ bn =
0 cannot hold. Thus the power seriesPis divergent for x = 1.
∞ n
When x = −1, the series becomes n=1 (−1) bn . Since bn > 1, we cannot have
limn→∞ (−1)n bn = 0, i.e., the power series is divergent for x = −1. Thus
interval of convergence is (−1, 1).

X (x + 3)n
b) We apply the Root Test to the series . Since limn→∞ (ln n)1/n = 1, we have
n=2
n ln n
that s¯ ¯
¯ n¯
n ¯ (x + 3) ¯ |x + 3|
ρ = lim ¯ ¯ = lim 1/n = |x + 3|.
n→∞ n ln n n→∞ n (ln n)1/n
The power series converges absolutely if ρ = |x + 3| < 1, and diverges if |x + 3| > 1.
The end points |x + 3| = 1, that is P
x = −2 and x = −4 must be checked independently.
When x = −2, the series becomes ∞ 1
n=2 n ln n . We can use integral test for this series
1
and see that it is divergent, because the function f (x) = x ln x
is positive, continuous
and decreasing for x ≥ 2, and
Z ∞ Z ∞
1 u=ln x du
dx = is divergent.
2 x ln x ln 2 u
P (−1)n
When x = −4, the series becomes ∞ n=2 n ln n . This series is an alternating series with
1
un = n ln n
. As n increases, {n ln n} increases and so {un } decreases. limn→∞ un =
P (−1)n
limn→∞ n ln n = 0. So by the Alternating Series Test, the series ∞
1
n=2 n ln n is conver-
gent.
So the interval of convergence is [−4, −2).
9. Find the sum of the following series and indicate the set of all x in which, your sum
formula

is true. ∞
X n2 xn X n
a) b) xn
n=1
n! n=1
n + 1


X xn
Solution. a) We know = ex , −∞ < x < ∞.
n=0
n!

X n xn−1
Take derivative: = ex , −∞ < x < ∞.
n=0
n!

X n xn
Multiply by x: = xex , −∞ < x < ∞.
n=0
n!
X∞
n2 xn−1
Take derivative: = ex + xex , −∞ < x < ∞.
n=0
n!

X n 2 xn
Multiply by x: = xex + x2 ex , −∞ < x < ∞.
n=0
n!
Since the term of the series for n = 0 is zero, we have that

X ∞
X
n 2 xn n 2 xn
= = (x + x2 ) ex , −∞ < x < ∞.
n=1
n! n=0
n!
P∞ 1
b) We use n=0 xn = 1−x
, −1 < x < 1. Let

X n 1 2 3 n
f (x) = xn = x + x2 + x3 + · · · + xn + · · · .
n=1
n+1 2 3 4 n+1

Then
1 2 2 3 3 4 n
x f (x) = x + x + x + ··· + xn+1 + · · ·
2 3 4 n+1
d
(x f (x)) = x + 2x2 + 3x3 + · · · + nxn + · · ·
dx
d
(x f (x)) = x(1 + 2x + 3x2 + · · · + nxn−1 + · · ·)
dx
d d
(x f (x)) = x (x + x2 + x3 + · · · + xn + · · ·)
dx dx µ ¶
d d 1 x x
(x f (x)) = x −1 = 2
= .
dx dx 1 − x (1 − x) (x − 1)2

So for −1 < x < 1,


Z Z µ ¶
x 1 1
x f (x) = dx = + dx
(x − 1)2 x − 1 (x − 1)2
1 1
= ln |x − 1| − + C = ln(1 − x) + + C.
x−1 1−x
To find C, we substitute x = 0, and find C = −1. So
1 x
x f (x) = ln(1 − x) + − 1 = ln(1 − x) + .
1−x 1−x
Then ∞ ½
X n ln(1−x)
+ 1
if − 1 < x < 1, x 6= 0
xn = f (x) = x 1−x

n=1
n+1 0 if x = 0.

10. a) Let f (x) be a continuous, decreasing


R ∞ and positive function on the interval [1, ∞),
and assume that the improper integral 1 f (x) dx is convergent. Let an = f (n). Show
that for every natural number N ,

X Z ∞
an ≤ f (x) dx.
n=N +1 N

P∞ 1
b) Assume that we want to find the sum s of the series n=1 n2 approximately, and we
estimate
1000
X 1
s ≈ s1000 = 2
.
n=1
n
Find an upper bound for the error in the above approximation.

Solution. a) Since the conditions


P∞ of the Integral Test are satisfied, we can use this test
and conclude
P that the series n=1 an converges. Thus for every natural number N , the
series ∞n=N +1 an also converges. We consider the graph of a typical such f (x). Since the
function is decreasing, we have that

X
an = total area of the rectangles
n=N +1
≤ area under the graph
Z ∞
= f (x) dx.
N

y = f (x)
area = aN +1

area = aN +2

area = aN +3

area = aN +4

area = an

O ··· ···
N N +1 N +2 N +3 N +4 ··· n−1 n ··· x
P
b) If we estimate s ≈ s1000 , then the error in this approximation isR ∞ 1
n=1001 n2 . Since the
1 ∞ 1
function f (x) = x2 is continuous, decreasing, positive on [1, ∞) and 1 x2 dx is convergent,
we can use part a) and get

X∞ Z ∞ Z c
1 1 1
0 < error = 2
≤ 2
dx = lim dx
n 1000 x c→∞ 1000 x2
n=1001
µ ¶¯c µ ¶
1 ¯¯ 1 1 1
= lim − ¯ = lim − + = .
c→∞ x 1000 c→∞ c 1000 1000

You might also like